ระยะทางสถิติระหว่างเหรียญที่สม่ำเสมอและลำเอียง


9

ปล่อย Uจะมีการกระจายชุดมากกว่าบิตและให้จะกระจายมากกว่าบิตที่บิตมีความเป็นอิสระและแต่ละบิตเป็นกับความน่าจะเป็น1จริงหรือไม่ที่ระยะทางทางสถิติระหว่างและคือเมื่อ ?nDn11/2ϵDUΩ(ϵn)n1/ϵ2


2
ใช่. ระยะทางทางสถิติระหว่างและเป็นอย่างน้อยซึ่งคือ ; ดูเช่นคำตอบของ matus ที่นี่: cstheory.stackexchange.com/questions/14471/…UVPrU(xi>n/2)PrD(xi>n/2)Ω(εn)
Yury

2
ขอบคุณ อาจอธิบายวิธีรับสิ่งนี้จากสิ่งที่ matus เขียนไว้ในคำตอบที่ฉันยอมรับ
มนู

อาจเป็นไปได้ที่มีประโยชน์: cstheory.stackexchange.com/q/22328/5038 , stats.stackexchange.com/q/17405/2921
DW

1
คุณสามารถทำได้ดีกว่าความไม่เท่าเทียมของ Slud ดู (2.13,2.14) ในarxiv.org/abs/1606.08920
Aryeh

คำตอบ:


7

แสดงถึงบิตสุ่มโดย x1,,xn. ตามคำนิยามระยะทางสถิติระหว่างU และ D อย่างน้อยก็ PrU(xit)PrD(xit)สำหรับทุกๆทีเราเลือก{n}tt=n/2+n

โปรดทราบว่าสำหรับบางอย่างต่อเนื่องแน่นอน0 ถ้าระยะทางสถิตินั้นอย่างน้อยและเราเสร็จแล้ว ดังนั้นเราคิดว่าด้านล่าง 2PrU(xit)c1c1>0PrD(xit)c1/2c1/2PrD(xit)c1/2

ให้สำหรับ IID Bernoulli ตัวแปรสุ่มกับ1/2-s เป้าหมายของเราคือการพิสูจน์ว่า{n}) โดยทฤษฎีบทค่าเฉลี่ย, สำหรับบางvarepsilon) ตอนนี้เราจะพิสูจน์ว่า ; นั่นหมายความว่าระยะทางสถิติที่ต้องการนั้นอย่างน้อยตามที่ต้องการf(s)=Pr(xit)x1,,xnPr(xi=1)=1/2sf(0)f(ε)=Ω(εn)

f(0)f(ε)=εf(ξ),
ξ(0,ε)f(ξ)Ω(n)Ω(nε)

เขียน, และ โปรดทราบว่า ดังนั้น,

f(ξ)=kt(nk)(12ξ)k(12+ξ)nk,
f(ξ)=kt(nk)(k(12ξ)k1(12+ξ)nk+(nk)(12ξ)k(12+ξ)nk1)=kt(nk)(12ξ)k(12+ξ)nkk/2+kξ(nk)/2+(nk)ξ(1/2ξ)(1/2+ξ).
k/2+kξ(nk)/2+(nk)ξ(1/2ξ)(1/2+ξ)=(2kn)/2+nξ(1/2ξ)(1/2+ξ)2(2tn)=4n.
f(ξ)4nkt(nk)(12ξ)k(12+ξ)nk=4nf(ξ)4nf(ε)4n(c1/2).
ที่นี่เราใช้สมมติฐานที่ว่า 2 เราแสดงให้เห็นว่า{n})f(ε)=PrD(x1++xnt)c1/2f(ξ)=Ω(n)

5

ประถมศึกษามากขึ้นและหลักฐานที่ messier เล็กน้อย (หรืออย่างน้อยก็ให้ฉัน)

เพื่อความสะดวกให้เขียนโดยใช้โดยสันนิษฐานε=γnγ[0,1)

เราลดขอบเขตการแสดงออกของ : dTV(P,U)

2dTV(P,U)=x{0,1}n|(12+γn)|x|(12γn)n|x|12n|=12nk=0n(nk)|(1+2γn)k(12γn)nk1|12nk=n2+nn2+2n(nk)|(1+2γn)k(12γn)nk1|Cnk=n2+nn2+2n|(1+2γn)k(12γn)nk1|
โดยที่เป็นค่าคงที่สัมบูรณ์ เรา จำกัด ขอบเขตการสรุปแต่ละครั้งแยกกัน: แก้ไขและเขียน , เพื่อให้แต่ละ summand ถูก จำกัด โดยปริมาณที่รวมกัน (เมื่อ ) ถึงC>0k=kn2[n,2n]
(1+2γn)k(12γn)nk=(14γ2n)n/2(1+2γn12γn)(14γ2n)n/2(1+2γn12γn)nne4γ2γ2
ne4γ2γ21>4γ2γ2>2γ ; หมายความว่าแต่ละคนเป็นแกมมา) สรุปแล้วสิ่งนี้จะทำให้ ตามที่อ้างสิทธิ์Ω(γ)
2dTV(P,U)Cnk=n2+nn2+2nΩ(γ)=Ω(γ)=Ω(εn)

(การใช้ Hellinger เป็นพร็อกซีเนื่องจากคุณสมบัติที่ดีของผลิตภัณฑ์การกระจาย wrt ดึงดูดและจะเร็วมาก แต่จะมีการสูญเสียจากปัจจัยกำลังสองในขอบเขตล่างสุด)
Clement C.

1
ดี! ฉันชอบวิธีการประถม เราควรจะสามารถทำให้มันไม่ใช่แบบอะซิมโทติคในเช่นกัน .... วิธีหนึ่งคือใช้แล้วใช้ไม่เท่าเทียมกันดี2/2} เมสเซอร์กว่านิดหน่อย n(1+z1z)n(1+2z)n1+weww2/2
usul
โดยการใช้ไซต์ของเรา หมายความว่าคุณได้อ่านและทำความเข้าใจนโยบายคุกกี้และนโยบายความเป็นส่วนตัวของเราแล้ว
Licensed under cc by-sa 3.0 with attribution required.